FAR practice exam

Réussis tes devoirs et examens dès maintenant avec Quizwiz!

Name 2 times where change in accounting principles would be used

change in rev recog, change in inventory valuation A change in the method of accounting for long-term contracts is a special change in accounting principle recognized on a retrospective basis. A change from the LIFO inventory method to any other method is a change in accounting principle that requires recognition on a retrospective basis (FASB ASC 250-10-45). Retro, prior period adjustment, RE adjusted

Selected information from the accounts of Row Co. on December 31, 20X1, follows: Total income since incorporation $420,000 Total cash dividends paid 130,000 Total value of property dividends distributed 30,000 Excess of proceeds over cost of treasury stock sold, accounted for using cost method 110,000 In its December 31, 20X1, financial statements, what amount should Row report as retained earnings? A. $260,000 B. $290,000 C. $370,000 D. $400,000

A. $260,000 income 420k - cash div paid 130k - property div 30k= 260k

Birk Co. purchased 30% of Sled Co.'s outstanding common stock on December 31, 20X1, for $200,000. On that date, Sled's stockholders' equity was $500,000, and the fair value of its identifiable net assets was $600,000. On December 31, 20X1, what amount of goodwill should Birk attribute to this acquisition? A. $0 B. $20,000 C. $30,000 D. $50,000

B. $20,000 When a company is bought, in whole or in part, the purchase price may exceed the fair values of all the company's net assets. The amount of this excess is goodwill from the purchase. Purchase cost of stock $200,000 -30% of identifiable assets (30%*$600k) 180,000 ------- Excess of purchase price over fair value of assets (Goodwill) $ 20,000

Packet Corp. is in the process of preparing its financial statements for the year ended December 31, 20X1. How would a gain on remeasuring a foreign subsidiary's financial statements from the local currency into the functional currency that occurred during 20X1 be classified in these financial statements? A. Income from continuing operations, with no separate disclosure B. Income from continuing operations, with separate disclosure (either on the face of statement or in the notes) C. Extraordinary items D. None of the answer choices are correct.

B. Income from continuing operations, with separate disclosure (either on the face of statement or in the notes) " It is also necessary to recognize currently in income all exchange gains and losses from remeasurement of monetary assets and liabilities."

Basic earnings per share for income from continuing operations and for net income are reported: A. on the face of the income statement. B. in the notes to the financial statements. C. for the current period only. D. if diluted earnings per share are presented.

Basic EPS is reported on the face of the income statement.

A not-for-profit voluntary health and welfare entity received a $500,000 permanent endowment. The donor stipulated that the income must be used for a mental health program. The endowment fund reported $60,000 net decrease in market value and $30,000 investment income. The organization spent $45,000 on the mental health program during the year. What amount of change in temporarily restricted net assets should the organization report? A. $75,000 decrease. B. $15,000 decrease. C. $0 D. $425,000 increase.

C. $0 Temp restricted CANNOT be negative (see page F9-44 section D.2.a.) and we can't spend anything out of Perm restricted FASB ASC 958-225-45-1 through 45-8 requires that all expenses be reported in the unrestricted category. Only net assets released from restriction reduce temporarily restricted revenue. The $30,000 investment income would increase temporarily restricted net assets and the release-from-restriction reclassification would reduce it $30,000. The $15,000 additional amount spent would come from unrestricted resources. The decrease in market value would affect only the endowment fund (classified as permanently restricted). Unrestricted, Temporarily Restricted, Permanently Restricted TOTAL Investment Income $30,000. $30,000 Release restriction $30,000 (30,000) 0 Expenses (45,000) 0 (45,000) Subtotal (15,000) 0 (15,000)

In preparing consolidated financial statements of a U.S. parent company with a foreign subsidiary, the foreign subsidiary's functional currency is the currency: A. in which the subsidiary maintains its accounting records. B. of the country in which the subsidiary is located. C. of the country in which the parent is located. D. of the environment in which the subsidiary primarily generates and expends cash.

D. of the environment in which the subsidiary primarily generates and expends cash. FASB ASC 830-10-45-2 states that the functional currency is the currency of the primary economic environment in which the entity operates or the currency in which most of the subsidiary's transactions are denominated.

The diluting effect of options and warrants and their equivalents is reflected in diluted EPS by application of the treasury stock method, which assumes that: A. proceeds from exercise are used to retire treasury stock. B. proceeds from exercise are used to issue treasury stock. C. proceeds from exercise are used to retire convertible debentures that were issued at par. D. proceeds from exercise are used to purchase common stock at the average market price.

D. proceeds from exercise are used to purchase common stock at the average market price. Exercise of options/warrants is assumed at the beginning of the period. Proceeds are assumed used to purchase common stock at the average market price during the period. The incremental shares are included in the denominator of diluted EPS.

Lys City records a compensated absences liability as a general government liability. The salary rate used to calculate the liability should normally be the rate in effect: A. when the unpaid compensated absences were earned. B. when the compensated absences are to be paid. C. at the balance sheet date. D. when the compensated absences were earned or are to be paid, or at the balance sheet date, whichever results in the lowest amount.

C. at the balance sheet date. GASB C60.107 indicates that a government's compensated absences liability normally "should be calculated based on the pay or salary rates in effect at the balance sheet date." (There is an exception for cases in which a justifiable lower amount should be used; for example, if the amount is based on a contractual agreement.)

FASB ASC 505-50-15-2 establishes a fair value approach for stock-based employee compensation plans. The fair value methodology is also extended to cover issuance of: A. bonds for certain types of assets. B. cash dividends to shareholders. C. equity instruments for goods and services. D. long-term debt for goods and services.

C. equity instruments for goods and services. All share-based payment transactions in which an entity acquires goods or services by issuing (or offering to issue) its shares, share options, or other equity instruments or by incurring liabilities to a goods or service provider that is not an employee in amounts based, at least in part, on the price of the entity's shares or other equity instruments or that require or may require settlement by issuing the entity's equity shares or other equity instruments.

On December 12, 20X1, Imp Co. entered into three forward exchange contracts, each to purchase 100,000 francs in 90 days. The relevant exchange rates are as follows: Forward Rate Spot Rate (for March 12, 20X2) --------- -------------------- December 12, 20X1 $.88 $.90 December 31, 20X1 .98 .93 Imp entered into the second forward contract to hedge a commitment to purchase equipment being manufactured to Imp's specifications. At December 31, 20X1, what amount of net foreign currency transaction gain should Imp include in income from this forward contract? A. $0 B. $3,000 C. $5,000 D. $10,000

A. $0 The "second" forward contract was entered into to hedge a purchase commitment, therefore (assuming that all other conditions are met) it qualifies as a hedge of a purchase commitment. In that case, the forward contract qualifies as a fair value hedge (rather than a cash flow hedge). Therefore, the change in fair value of the derivative (the forward contract) should be included in net income, as should the change in fair value of the hedged commitment. Accordingly, the $3,000 gain (100,000 francs × ($.90-$.93)) would be included in income from continuing operations. If the hedge associated with the derivative qualified as a cash flow hedge, the unrealized gain would be included in other comprehensive income rather than in net income. However, the increase in the fair value of the hedged item, the purchase commitment, should be a $3,000 loss. The unrealized gain of $3,000 associated with the derivative (the hedging instrument) and the $3,000 unrealized loss associated with the hedged instrument (the purchase commitment) should result in a net unrealized gain/loss of $0. Thus, the net unrealized gain/loss included in income from continuing operations is $0.

A company decided to sell an unprofitable division of its business. The company can sell the entire operation for $800,000, and the buyer will assume all assets and liabilities of the operations. The tax rate is 30%. The assets and liabilities of the discontinued operation are as follows: Buildings $5,000,000 Accumulated depreciation 3,000,000 Mortgage on buildings 1,100,000 Inventory 500,000 Accounts payable 600,000 Accounts receivable 200,000 What is the after-tax net loss on the disposal of the division? A. $140,000 B. $200,000 C. $1,540,000 D. $2,200,000

A. $140,000 1 mil total - 800k sales price = 200k loss 200k * 30% tax = 60k tax 200-60 = 140k after tax

Name 5 times where change in accounting estimate would be used

change in asset life, change in salvage value, change in depreciation, change in warranty cost, change in ADA A change in estimated asset life is a change in an accounting estimate that is recognized on a prospective basis (FASB ASC 250-10-45). A change in estimated warranty cost is a change in an accounting estimate, which is recognized on a prospective basis (FASB ASC 250-10-45). The change in depreciation methods is a change in accounting estimate effected by a change in accounting principle and would be recognized under the prospective approach (FASB ASC 250-10-45). Prospective

During the previous year, Yvo Corp. installed a production assembly line to manufacture furniture. In the current year, Yvo purchased a new machine and rearranged the assembly line to install this machine. The rearrangement did not increase the estimated useful life of the assembly line, but it did result in significantly more efficient production. The following expenditures were incurred in connection with this project: Machine $75,000 Labor to install machine 14,000 Parts added in rearranging the assembly line to provide future benefits 40,000 Labor and overhead to rearrange the assembly line 18,000 What amount of the above expenditures should be capitalized in the current year? A. $147,000 B. $107,000 C. $89,000 D. $75,000

A. $147,000 When a rearrangement adds to the overall efficiency of the production process for equipment, that is more than simply a repair. The rearrangement will benefit the machine over its remaining life and should be capitalized into the cost of the asset. $75,000 + $14,000 + $40,000 + $18,000 = $147,000

On January 2, 20X1, Blake Co. sold a used machine to Cooper, Inc., for $900,000, resulting in a gain of $270,000. On that date, Cooper paid $150,000 cash and signed a $750,000 note bearing interest at 10%. The note was payable in three annual installments of $250,000 beginning January 2, 20X2. Blake appropriately accounted for the sale under the installment method. Cooper made a timely payment of the first installment on January 2, 20X2, of $325,000 which included accrued interest of $75,000. What amount of deferred gross profit should Blake report on December 31, 20X2? A. $150,000 B. $172,500 C. $180,000 D. $225,000

A. $150,000 For installment sales, one must compute the gross profit percentage as a percentage of installment sales revenue for each year. The deferred gross profit for a year is the gross profit percentage for the year multiplied by the remaining uncollected installment sale receivables for the year. Gross profit rate = $270,000 ÷ $900,000 = 30% Cash collected prior to December 31, 20X2: January 2, 20X1 $150,000 January 2, 20X2 250,000 -------- Total $400,000 ======== Cash remaining to be collected = $900,000 - $400,000 = $500,000 Deferred gross profit to be reported on December 31, 20X2 = 30% × $500,000 = $150,000

Pine Corp.'s books showed pretax income of $800,000 for the current year ended December 31. In the computation of federal income taxes, the following data were considered: Gain on involuntary conversion (Pine has elected to replace the property within the statutory period using total proceeds.) $350k Depreciation deducted for tax purposes in excess of depreciation deducted for book purposes $50k Federal estimated tax payments $70k Enacted federal tax rates 30% What amount should Pine report as its current federal income tax liability on its December 31 balance sheet? A. $50,000 B. $65,000 C. $120,000 D. $135,000

A. $50,000 The current income tax liability is based on the taxable income for the year and the tax rate for the year, so one needs to compute taxable income first. The pretax income was $800,000 and part of this amount is a gain that qualifies for tax deferral, the reinvested proceeds from the involuntary conversion gain. Also, the amount of tax depreciation is used to compute taxable income. Taxable income is $400,000 ($800,000 less the gain of $350,000, and less the additional tax depreciation of $50,000). This amount multiplied by the tax rate of 30% gives us the tax due of $120,000 ($400,000 × 0.30). Because some of this amount due has been paid in estimates already, only the remaining $50,000 is a liability yet to pay ($120,000 - $70,000).

Tam Co. reported the following items in its year-end financial statements: Capital expenditures $1,000,000 Capital lease payments 125,000 Income taxes paid 325,000 Dividends paid 200,000 Net interest payments 220,000 What amount should Tam report as supplemental disclosures in its statement of cash flows prepared using the indirect method? A. $545,000 B. $745,000 C. $1,125,000 D. $1,870,000

A. $545,000 interest tax paid + net interest pmt Regardless of whether the direct or indirect method is used to determine cash flows from operating activities, the following items are required to be disclosed: Amount of income taxes paid during the period ($325,000) Amount of interest paid during the period ($220,000) $325,000 + $220,000 = $545,000

Cado Co.'s payroll for the month ended January 31 is summarized as follows: Total wages $100,000 Amount of wages subject to payroll taxes: FICA 80,000 Unemployment 20,000 Payroll tax rates: FICA for employer and employee 7% each Unemployment 3% In its January 31 balance sheet, what amount should Cado accrue as its share of payroll taxes? A. $6,200 B. $10,000 C. $11,800 D. $17,000

A. $6,200 Employer's payroll taxes do not include amounts withheld from employee pay. Employer payroll taxes include: the employer's matching share of FICA taxes ($80,000 × 0.07 = $5,600) and unemployment taxes ($20,000 × 0.03 = $600), for a total of $6,200 ($5,600 + $600).

On August 1 of the current year, Kern Company leased a machine to Day Company for a 6-year period requiring payments of $10,000 at the beginning of each year. The machine cost $48,000, which is the fair value at the lease date, and has an estimated life of eight years with no residual value. Kern's implicit interest rate is 10% and present value factors are as follows: Present value of an annuity due of $1 at 10% for 6 periods: 4.791 Present value of an annuity due of $1 at 10% for 8 periods: 5.868 Kern appropriately recorded the lease as a direct-financing lease. At the inception of the lease, the gross lease receivables account balance should be: A. $60,000. B. $58,680. C. $48,000. D. $47,910.

A. $60,000. When one is recognizing a lease transaction as a capital lease for the lessor, one has an account for the gross receivable under the lease. This account keeps track, in undiscounted fashion, of the entire lease rental payments to be received by the lessor. Here the gross receivables balance at the beginning of the lease is all six payments of $10,000, or $60,000.

On December 31, 20X1, Date Co. awaits judgment on a lawsuit for a competitor's infringement of Date's patent. Legal counsel believes it is probable that Date will win the suit and indicated the most likely award together with a range of possible awards. How should the lawsuit be reported in Date's 20X1 financial statements? A. In note disclosure only B. By accrual for the most likely award C. By accrual for the lowest amount of the range of possible awards D. Neither in note disclosure nor by accrual

A. In note disclosure only If Date Co. wins the lawsuit, the award paid to Date will be a gain. FASB ASC 450-30-50-1 provides that gain contingencies should not be reflected in the accounts (i.e., accrued) but that adequate disclosure should be made in notes to the financial statements. A loss contingency would be reported by accrual for the most likely award or for the lowest amount of the range of possible awards if no amount can be considered most likely. (FASB ASC 450-20-25-4)

On November 2, 20X1, Finsbury, Inc., issued warrants to its stockholders giving them the right to purchase additional $20 par value common shares at a price of $30. The stockholders exercised all warrants on March 1, 20X2. The shares had market prices of $33, $35, and $40 on November 2, 20X1, December 31, 20X1, and March 1, 20X2, respectively. What were the effects of the warrants on Finsbury's additional paid-in-capital and net income? A. Additional paid-in capital increased in 20X2; no effect on net income. B. Additional paid-in capital increased in 20X1; no effect on net income. C. Additional paid-in capital increased in 20X2; net income decreased in 20X1 and 20X2. D. Additional paid-in capital increased in 20X1; net income decreased in 20X1 and 20X2.

A. Additional paid-in capital increased in 20X2; no effect on net income. The only accounting effect of the issue and exercise of the warrants would be a memo entry to record the issuance of warrants on November 2, 20X1, and the following journal entry to record the exercise of warrants on March 1, 20X2: Cash XX Common stock XX Additional paid-in capital XX (We know there was additional paid-in capital because the exercise price was $30, $10 more than the par value of the stock.) As a result, additional paid-in capital increased in 20X2, but net income was not affected.

A corporation entered into a purchase commitment to buy inventory. At the end of the accounting period, the current market value of the inventory was less than the fixed purchase price, by a material amount. Which of the following accounting treatments is most appropriate? A. Describe the nature of the contract in a note to the financial statements, recognize a loss in the income statement, and recognize a liability for the accrued loss B. Describe the nature of the contract and the estimated amount of the loss in a note to the financial statements, but do not recognize a loss in the income statement C. Describe the nature of the contract in a note to the financial statements, recognize a loss in the income statement, and recognize a reduction in inventory equal to the amount of the loss by use of a valuation account D. Neither describe the purchase obligation nor recognize a loss on the income statement or balance sheet

A. Describe the nature of the contract in a note to the financial statements, recognize a loss in the income statement, and recognize a liability for the accrued loss

Estimates are a necessary part of the preparation of financial statements. It is necessary to explicitly communicate to the users of the financial statements that estimates have been used and that many of the amounts reported are approximations rather than exact amounts. Which of the following is not an example of certain significant estimates as listed in Topic 275 of the FASB's Accounting Standards Codification? A. Expensed computer software costs B. Inventory subject to rapid technological obsolescence C. Contingent liabilities for obligations of other entities D. Estimated net proceeds recoverable, the provisions for expected loss to be incurred, or both, on disposition of a business or assets

A. Expensed computer software costs Examples of items that might require disclosure under this topic include the following (FASB ASC 275-10-50-15): Inventory subject to rapid technological obsolescence Specialized equipment subject to technological obsolescence Environmental remediation-related obligations Contingent liabilities for obligations of other entities Amounts reported for long-term obligations, such as amounts reported for pension and postemployment benefits Estimated net proceeds recoverable, the provisions for expected loss to be incurred, or both, on disposition of a business or assets

Which of the following circumstances would result in a deferred tax asset for the current year? A. Expenses that are recognized in financial income this year and deductible next year B. Expenses that are deductible this year and recognized in financial income next year C. Revenues that are recognized in financial income this year and taxable next year D. Revenues that are recognized in financial income this year but are not subject to taxation

A. Expenses that are recognized in financial income (not taxable) this year and deductible next year -taxable recog next year A deferred tax liability or asset is recognized for the estimated future tax effects attributable to temporary differences and carryforwards. Deferred tax assets (DFA) are created when taxes are paid or carried forward but not yet recognized in the income statement. For example, DTAs are created when expenses (not revenues) are recognized in book income (i.e., financial income) before they are recognized in tax income, such as a loss carryover from the prior year. DTAs also arise when an entity incurs an accounting expense such as bad-debt write-offs, but does not recognize the expense for tax periods until a future period.

At the end of the year, Town City's general fund included, among the assets, an inventory of office supplies worth $1,500 and prepaid fire insurance covering the first two months of the subsequent year of $600. As a result of these assets, the fund balance presented in the balance sheet of the general fund would include which of the following? A. Fund balance—nonspendable: $2,100 B. Fund balance—nonspendable: $1,500; Fund balance—committed: $600 C. Fund balance—assigned: $1,500; Fund balance—restricted: $600 D. Fund balance—restricted: $2,100

A. Fund balance—nonspendable: $2,100 The portion of fund balance that reflects equity for amounts that cannot be spent because they are not in spendable form would be termed "nonspendable." Both supplies inventory and prepaid expenses are not spendable. Committed fund balance would result from an action of the highest level of the government and assigned fund balance would reflect action by a government designee. Restrictions of fund balance would reflect specific purposes externally imposed by creditors, grantors, or enabling legislation.

GASB I60, Investments—Securities Lending, states that a government that (1) loans securities to a broker-dealer and (2) receives collateral in the form of other securities that the government cannot pledge or sell without borrower default should report: the securities lent as assets. the collateral received as assets. a liability for the government's obligation to return the collateral securities. A. I only B. I and III C. II and III D. I, II, and III

A. I only The GASB Codification (Section I60.103) states: "Governmental entities should report securities lent (the underlying securities) as assets in their balance sheets." Further, GASB I60.105 states that "securities lending transactions collateralized by letters of credit or by securities that the governmental entity does not have the ability to pledge or sell unless the borrower defaults should not be reported as assets and liabilities in the balance sheet."

Which of the following subsequent events must be recognized in the financial statements? A. Loss on an uncollectible trade account receivable as a result of a customer's deteriorating financial condition leading to bankruptcy after the balance sheet date but before the financial statements are issued or are available to be issued B. Sale of a bond or capital stock issued after the balance sheet date but before financial statements are issued or are available to be issued C. Changes in the fair value of assets or liabilities (financial or nonfinancial) or foreign exchange rates after the balance sheet date but before financial statements are issued or are available to be issued D. Settlement of litigation when the event giving rise to the claim took place after the balance sheet date but before financial statements are issued or are available to be issued

A. Loss on an uncollectible trade account receivable as a result of a customer's deteriorating financial condition leading to bankruptcy after the balance sheet date but before the financial statements are issued or are available to be issued The correct answer is "loss on an uncollectible trade account receivable as a result of a customer's deteriorating financial condition leading to bankruptcy after the balance sheet date but before the financial statements are issued or are available to be issued." An entity must recognize in the financial statements the effects of all subsequent events that provide additional evidence about conditions that existed at the date of the balance sheet. The other answer choices are incorrect because an entity must not recognize subsequent events that provide evidence about conditions that did not exist at the date of the balance sheet but arose after the balance sheet date but before financial statements are issued or are available to be issued.

Bay Manufacturing Co. purchased a 3-month U.S. Treasury bill. In preparing Bay's statement of cash flows, this purchase would: A. have no effect. B. be treated as an outflow from financing activities. C. be treated as an outflow from investing activities. D. be treated as an outflow from operating activities.

A. have no effect. FASB ASC 230-10-20 focuses on cash and cash equivalents. The following explanation is offered: Examples of items commonly considered to be cash equivalents are Treasury bills, commercial paper, money market funds, and federal funds sold (for an enterprise with banking operations). Thus, the purchase of U.S. Treasury bills would have no effect on the statement of cash flows.

A major exception to the general rule of expenditure accrual for governmental units relates to unmatured: A. principal of general long-term debt and interest on general long-term debt. B. principal of general long-term debt. C. interest on general long-term debt. D. neither principal of general long-term debt nor interest on general long-term debt.

A. principal of general long-term debt and interest on general long-term debt. Governmental units use a modified accrual basis of accounting. Expenditures should be recorded as fund liabilities are incurred or assets are expended. However, the expenditure rules will not apply to the principal or to the interest on debt as specified by the GASB.

Selected information from the accounting records of Dalton Manufacturing Company is as follows: Net sales from the current yr $1,800,000 COGS for the current yr 1,200,000 Inventories at 12/31 previous year 336,000 Inventories at 12/31 current year 288,000 Assuming that there are 300 working days per year, what is the number of days' sales in average inventories for the current year? Assuming that there are 300 working days per year, what is the number of days' sales in average inventories for the current year? A. 78 B. 72 C. 52 D. 48

A. 78 Days' sales in inventory = (Average inventory ÷ Cost of goods sold) × Working days per year: (($336,000 + $288,000) / 2) / $1,200,000 = 0.26 0.26 × 300 days = 78 days

Jent Corp. purchased bonds at a discount of $10,000. Subsequently, Jent sold these bonds at a premium of $14,000. During the period that Jent held this investment, amortization of the discount amounted to $2,000. What amount should Jent report as gain on the sale of bonds? A. $12,000 B. $22,000 C. $24,000 D. $26,000

B. $22,000 14k - (-10)k - 2k depreciation = 22k To do the calculation, use a number as the bonds face value, say $100,000. If such a bond was purchased at a discount of 10,000, that would be a carrying value of $90,000 ($100,000 - $10,000). If, while the bonds were held, $2,000 of discount was amortized, then the carrying value of the bonds would be $92,000, as the discount amortized adds to the bond carrying value, so that the carrying value approaches face value of $100,000. If the bonds were then sold to another investor at a premium of $14,000, then the new investor would pay face value plus $14,000 for a total of $114,000 ($100,000 + $14,000). When an investment with a carrying value of $92,000 is sold for $114,000, a gain of the difference is recognized: $114,000 - $92,000 = $22,000

Leer Corp.'s pretax income in the current year was $100,000. The temporary differences between amounts reported in the financial statements and the tax return are as follows: Depreciation in the financial statements was $8,000 more than tax depreciation. The equity method of accounting resulted in financial statement income of $35,000. A $25,000 dividend was received during the year, which is eligible for the 80% dividends-received deduction. Leer's effective income tax rate was 30%. In its current year income statement, Leer should report a current provision for income taxes of: A. $26,400. B. $23,400. C. $21,900. D. $18,600.

B. $23,400. The current provision for income taxes is simply the taxable income for the year multiplied by the tax rate for the year. Thus, we need to find the taxable income for the year. Start with the pretax income of $100,000, and add $8,000 to it because tax depreciation expense was less than book depreciation. The $35,000 equity method income for financial accounting needs to be subtracted since it is not the taxed amount: $100,000 + $8,000 - $35,000 = $73,000 The dividends that are taxable are subject to a dividends-received deduction of 80%. Thus, only add in $5,000 of the dividends ($25,000 × 0.20 (1 − 0.80)), because the dividends, though taxable in part, are not financial accounting income when applying the equity method: $73,000 + $5,000 = $78,000 Thus, taxable income is $78,000 ($100,000 + $8,000 - $35,000 + $5,000) and the current income tax due is $23,400: $78,000 × 0.30 = $23,400

At the acquisition date, July 2, 20X1, reporting unit R has a fair value of $370,000 and a carrying amount (including goodwill of $100,000) of $470,000. On December 31, 20X1, the fair value of the assets and liabilities assigned to reporting unit R is $330,000, and the fair value of R is $400,000. The goodwill impairment loss reportable is: A. $0. B. $30,000. C. $40,000. D. $100,000.

B. $30,000. Step 1, Compare: (a) year-end fair value of reporting unit $400,000 (b) carrying amount, including goodwill $470,000 If (b) exceeds (a), go to step 2. Step 2, Compare: (a) implied fair value of reporting unit goodwill ($400,000 - $330,000) $ 70,000 (b) carrying amount of goodwill $100,000 Since (b) exceeds (a) by $30,000, an impairment loss of $30,000 is recognized. Note: The impairment loss cannot exceed the recorded goodwill.

Bake Co.'s trial balance included the following at December 31, 20X1: Accounts payable $ 80,000 Bonds payable, due 20X2 300,000 Discount on bonds payable 15,000 Deferred income tax liability 25,000 The deferred income tax liability is not related to an asset for financial accounting purposes and is expected to reverse in 20X2. What amount should be included in the current liability section of Bake's December 31, 20X1, balance sheet (statement of financial position)? A. $365,000 B. $390,000 C. $395,000 D. $420,000

B. $390,000 All of the liabilities are current liabilities because all will be paid within the next year. Accounts payable $ 80,000 Bonds payable 300,000 Discount on bonds payable (15,000) Deferred income tax liability 25,000 -------- Total $390,000

On January 2, 20X1, West Co. issued 9% bonds in the amount of $500,000, which mature on January 2, 20X8. The bonds were issued for $469,500 to yield 10%. Interest is payable annually on December 31. West uses the interest method of amortizing bond discount. In its June 30, 20X1, balance sheet, what amount should West report as bonds payable? A. $469,500 B. $470,475 C. $471,025 D. $500,000

B. $470,475 500,000 -469,500 = 30.5k diff 9% bond interest (500k) and 10% yield (469,500) = 975 500,000-469,500-975 = 470,475 The bonds were issued at a discount, and one must amortize the discount up to the balance sheet date. The interest paid is 9% of the face amount of $500,000, but the interest expense is the yield multiplied by the carrying amount. The difference of these is the discount amortization added to the carrying amount of the bonds. Face amount of bonds $500,000 Less: Issue price 469,500 -------- Bond discount at issue date $ 30,500 June 30, 20X1, JE to accrue interest and amortize discount: Dr. Cr. Bond interest expense $23,475 (10% x $469,500 x 6/12) Discount on bonds payable ($23,475 - $22,500) (PLUG) $ 975 Interest Payable (9% x $500,000 x 6/12) 22,500 Bonds payable on June 30, 20X1, balance sheet: Bonds payable $500,000 Less: Unamortized discount ($30,500 - $975) 29,525 -------- Net bonds payable $470,475

On July 1, 20X1, Ran County issued realty tax assessments for its fiscal year ending June 30, 20X2. On September 1, 20X1, Day Co. purchased a warehouse in Ran County. The purchase price was reduced by a credit for accrued realty taxes. Day did not record the entire year's real estate tax obligation, but instead recorded tax expenses at the end of each month by adjusting prepaid real estate taxes or real estate taxes payable, as appropriate. On November 1, 20X1, Day paid the first of two equal installments of $12,000 for realty taxes. What amount of this payment should Day have recorded as a debit to real estate taxes payable? A. $4,000 B. $8,000 C. $10,000 D. $12,000

B. $8,000 The payable has been accruing since July at the rate of $2,000 per month ($24,000 ÷ 12) and is at $8,000 when paid. (July - Oct = 4 months = 8k) (Nov - Dec = 2 months = 4k prepaid) Dr. Cr. Semi-annual realty tax payment = $12,000 Monthly tax accrual = $12,000 / 6 months = $2,000 September 1, 20X1, entry to purchase warehouse: Warehouse xxx Cash xx Real estate taxes payable $ 4,000 (2 months x $2,000) September 30, 20X1, entry to accrue taxes: Real estate taxes expense $2,000 Real estate taxes payable $ 2,000 October 31, 20X1, entry to accrue taxes: Same as September 30 November 1, 20X1, entry to pay taxes: Prepaid real estate taxes $4,000 (Nov/Dec) Real estate taxes payable 8,000 Cash $12,000 The November and December accruals should credit prepaid real estate taxes.

Hunt Co. purchased merchandise for 300,000 British pounds from a vendor in London on November 30, 20X1. Payment in British pounds was due on January 30, 20X2. The exchange rates to purchase one pound were as follows: 11/30/X1 12/31/X1 ---------- ---------- Spot-rate $1.65 $1.62 30-day rate 1.64 1.59 60-day rate 1.63 1.56 In its December 31, 20X1, income statement, what amount should Hunt report as foreign exchange gain? A. $12,000 B. $9,000 C. $6,000 D. $0

B. $9,000, spot rates FASB ASC 830-20 (Foreign Currency Transactions) provides that a gain or loss on a forward contract is computed by multiplying the foreign currency amount of the forward contract by the difference between the spot rate at the balance sheet date and the spot rate at the date of inception of the forward contract. Hunt's reported foreign exchange gain = 300,000 British pounds x ($1.65 - $1.62) per British pound = $9,000

A company issued a bond with a stated rate of interest that is less than the effective interest rate on the date of issuance. The bond was issued on one of the interest payment dates. What should the company report on the first interest payment date? A. An interest expense that is less than the cash payment made to bondholders B. An interest expense that is greater than the cash payment made to bondholders C. A debit to the unamortized bond discount D. A debit to the unamortized bond premium

B. An interest expense that is greater than the cash payment made to bondholders Interest expense = Carrying amount × Effective interest rate. Cash payment amount = Face amount × Stated interest rate. In this example, the stated rate is less than the effective rate, so the cash payment is less than the interest expense.

In which situation(s) should property taxes due to a governmental unit be recorded as deferred revenue? Property taxes receivable are recognized in advance of the year for which they are levied. Property taxes receivable are collected in advance of the year in which they are levied. A. I only B. Both I and II C. II only D. Neither I nor II

B. Both I and II GASB N50.115 states that governments should recognize property tax revenues in the period for which the taxes are levied even if a legal claim or actual payment occurs in a previous period. Property taxes are often levied in the fiscal year prior to the year in which they legally can be expended. To gain accounting control over the receivable, it should be recorded at the time of levy. However, the offsetting credit should be to a deferred revenue account since the revenue, although measurable, is not "available" until the following fiscal year. If previously levied, taxes collected in advance should be recorded with a debit to cash and a credit to taxes receivable. If not previously levied, the taxes collected should be recorded with a debit to cash and a credit to deferred revenues.

In a sale-leaseback transaction, a gain resulting from the sale should be deferred at the time of the sale-leaseback and subsequently amortized when: I. the seller-lessee has transferred substantially all the risks of ownership. II. the seller-lessee retains the right to substantially all of the remaining use of the property. A. I only B. II only C. Both I and II D. Neither I nor II

B. II only II. the seller-lessee retains the right to substantially all of the remaining use of the property. FASB ASC 840-40-25-4 provides that if the lease meets one of the criteria for capital lease treatment (it does—the lease transfers title to the seller-lessee at end of lease term), then any gain on the sale should be deferred and amortized. Again, the key element is the retention of the right to all remaining use of the property. Transfer of risks of ownership is not one of the four criteria for capital lease treatment.

Which of the following best describes the general disclosure principle? A. Certain information may be presented either on the face of the financial statements or in the notes to the financial statements. Disclosure in the notes to the financial statements is needed only when the information required to be disclosed is not displayed on the face of the financial statements and the MD&A. B. Certain information may be presented either on the face of the financial statements or in the notes to the financial statements. Disclosure in the notes to the financial statements is needed only when the information required to be disclosed is not displayed on the face of the financial statements. C. Disclosure in the notes to the financial statements is needed only when management feels it is necessary to supplement information presented on the face of the financial statements. D. Disclosure in the notes to the financial statements is needed only when the meaningful information is not provided elsewhere therein.

B. Certain information may be presented either on the face of the financial statements or in the notes to the financial statements. Disclosure in the notes to the financial statements is needed only when the information required to be disclosed is not displayed on the face of the financial statements. Per GASB 2300.103, certain information may be presented either on the face of the financial statements or in the notes to the financial statements. Disclosure in the notes to the financial statements is needed only when the information required to be disclosed is not displayed on the face of the financial statements. Disclosure in the MD&A or "elsewhere" are not alternatives for disclosure provided by the GASB Codification. Note disclosure includes "essential," not "supplemental," information that is not displayed on the face of the financial statements.

How would a municipality that uses modified accrual and encumbrance accounting record the transaction of the receipt of supplies from approved purchase orders and the approval of the related invoices? A. Debit encumbrances control. B. Debit expenditures control. C. Credit appropriations control. D. Credit other financing uses.

B. Debit expenditures control. Modified accrual and encumbrance accounting are features of governmental fund accounting. In this instance, the receipt of supplies and approval of the related invoices indicate that an expenditure and a liability should be recorded in the governmental fund which is to pay for the supplies. Any encumbrance entry previously recorded for the supplies should be reversed (credit encumbrance control)

A business combination is accounted for as an acquisition (initiated in a fiscal year beginning after December 15, 2008). Which of the following expenses related to the business combination should be included, in total, in the determination of net income of the combined corporation for the period in which the expenses are incurred? A. Fees of finders and consultants: Yes; Issuance fees for equity securities issues: Yes B. Fees of finders and consultants: Yes; Issuance fees for equity securities issues: No C. Fees of finders and consultants: No; Issuance fees for equity securities issues: Yes D. Fees of finders and consultants: No; Issuance fees for equity securities issues: No

B. Fees of finders and consultants: Yes; Issuance fees for equity securities issues: No (paid-in-capital account.) Business combinations accounted for as an acquisition should treat expenses related to the combination as follows: Out-of-pocket costs such as fees of finders and consultants are expensed. Issuance costs such as SEC filing fees are charged to the paid-in-capital account. Recognition Conditions To qualify for recognition as part of applying the acquisition method, the identifiable assets acquired and liabilities assumed must meet the definitions of assets and liabilities in FASB Concepts Statement No. 6, Elements of Financial Statements, at the acquisition date. For example, costs the acquirer expects but is not obligated to incur in the future to effect its plan to exit an activity of an acquiree or to terminate the employment of or relocate an acquiree's employees are not liabilities at the acquisition date. Therefore, the acquirer does not recognize those costs as part of applying the acquisition method. Instead, the acquirer recognizes those costs in its postcombination financial statements in accordance with other applicable generally accepted accounting principles (GAAP). Acquisition-Related Costs Acquisition-related costs are costs the acquirer incurs to effect a business combination. Those costs include finder's fees; advisory, legal, accounting, valuation, and other professional or consulting fees; general administrative costs, including the costs of maintaining an internal acquisitions department; and costs of registering and issuing debt and equity securities. The acquirer shall account for acquisition-related costs as expenses in the periods in which the costs are incurred and the services are received, with one exception. The costs to issue debt or equity securities shall be recognized in accordance with other applicable GAAP.

Which of the following activities should be excluded when governmental fund financial statements are converted to government-wide financial statements? A. Proprietary activities B. Fiduciary activities C. Government activities D. Enterprise activities

B. Fiduciary activities The government-wide financial statements display information about the reporting government as a whole. The statements would report the governmental activities reported in the governmental funds and the proprietary and enterprise activities accounted for in enterprise funds. The fiduciary activities are not considered part of the operations of the government itself and would not be included in the government-wide financial reports.

A not-for-profit voluntary health and welfare entity should report a contribution for the construction of a new building as cash flows from which of the following in the statement of cash flows? A. Operating activities B. Financing activities C. Capital financing activities D. Investing activities

B. Financing activities According to FASB ASC 958-230-55-3, a contribution to a not-for-profit restricted to long-term purposes like construction shall be reported as a cash flow from financing activities. Cash flows received from investment income restricted by donor stipulation to the same purposes also are reported as financing activities, not as operating activities.

Which of the following classifications is required for reporting of expenses by all not-for-profit entities? A. Natural classification in the statement of activities or notes to the financial statements B. Functional classification in the statement of activities or notes to the financial statements C. Functional classification in the statement of activities and natural classification in a matrix format in a separate document D. Functional classification in the statement of activities and natural classification in the notes to the financial documents

B. Functional classification in the statement of activities or notes to the financial statements Financial reporting for a not-for-profit should provide information about the service efforts of the entity. Therefore, the FASB Accounting Standards Codification requires expenses to be reported by functional classification (i.e., program services, management, fundraising, etc.). Only those not-for-profits that are voluntary health and welfare entities must augment the functional classification of expenses that appears in the statement of activities with a natural classification of expenses, displayed in a matrix format, that is shown in a separate document, a statement of functional expenses.

A company has available-for-sale investments that cost $50,000 and were valued at $45,000 at the beginning of the current period during which the investments were sold for $48,000. Which of the following best reflects the impact of these events on the elements of comprehensive income of the current year? A. Impact on net income: $3,000 gain; Other comprehensive income (reclassification): $5,000 loss; Comprehensive income: $2,000 loss B. Impact on net income: $2,000 loss; Other comprehensive income (reclassification): $5,000 gain; Comprehensive income: $3,000 gain C. Impact on net income: $4,000 gain; Other comprehensive income (reclassification): $6,000 loss; Comprehensive income: $2,000 loss D. Impact on net income: $5,000 loss; Other comprehensive income (reclassification): $5,000 gain; Comprehensive income: $3,000 gain

B. Impact on net income: $2,000 loss; Other comprehensive income (reclassification): $5,000 gain; Comprehensive income: $3,000 gain The relationship of net income and comprehensive income can be shown as follows: Other Comprehensive Net income + comprehensive = income income Ignoring any tax effects, these transactions and events affect comprehensive income as follows: Net income: Realized loss ($48,000 - $50,000) $(2,000) Other comprehensive income: Reclassification adjustment gain 5,000 -------- Comprehensive income $ 3,000 ======== A realized loss of $2,000 is recognized because investments costing $50,000 were sold for $48,000. That realized loss is included in net income. The $5,000 reclassification gain is required to offset the previously recognized unrealized loss ($50,000 - $45,000). FASB ASC 220-10-20 defines "other comprehensive income" as "all revenues, expenses, gains, and losses that under generally accepted accounting principles (GAAP) are included in comprehensive income but excluded from net income." Currently, existing GAAP specifies that unrealized holding gains and losses on available-for-sale securities should be reported as direct charges or credits to equity. Thus, these gains and losses constitute other comprehensive income. To prevent including certain items in the determination of comprehensive income twice, reclassification adjustments are required for a transaction or event that has been included as a component of other comprehensive income and later becomes a component of net income. Impact Impact Years Before Year of Sale Sale ------------ -------- Net income -0- $(2,000) Other comprehensive income: Unrealized gain/(loss) $(5,000) Reclassification -0- 5,000 -------- -------- Comprehensive income $(5,000) $ 3,000

Which of the following best describes a situation in which an unconditional contribution should be recognized as revenue by a private not-for-profit organization? A. In the period when cash or other assets are received at the carrying value on the books of the donor B. In the period received at fair value C. In the period in which the donor states its unconditional promise to make the contribution and at the carrying value on the books of the donor D. In the period in which the donor states its intention to make the contribution and at fair value

B. In the period received at fair value Unconditional contributions, whether promised or received as cash, are recognized as revenue in the period received. Contributions revenue should be measured at fair value, not donor's book value. Donor intentions to give, rather than unconditional promises, are not considered revenue.

On August 15 of the current year, Benet Co. sold goods for which it received a note bearing the market rate of interest on that date. The 4-month note was dated this July 15. Note principal, together with all interest, is due November 15. When the note was recorded on August 15, which of the following accounts increased? A. Unearned discount B. Interest receivable C. Prepaid interest D. Interest revenue

B. Interest receivable This note was received after it was made, and thus the note has already accrued interest. This interest that has already accrued, along with the rest of the interest that will be received, will be received by the holder of the note when it comes due. Only the interest that accrues to the holder of the note while they hold the note will count as interest revenue to them. Thus, the interest that accrued on the note prior to the point of acquiring the note is not interest revenue, but only interest receivable.

What are the components of the lease receivable for a lessor involved in a direct financing lease? A. The minimum lease payments plus any executory costs B. The minimum lease payments plus residual value C. The minimum lease payments less residual value D. The minimum lease payments less initial direct costs

B. The minimum lease payments plus residual value The lessor shall measure the gross investment in a direct financing lease initially as the sum of the following amounts: The minimum lease payments + unguaranteed residual value accruing to the benefit of the lessor

In Year 1, a company reported in other comprehensive income an unrealized holding loss on an investment in available-for-sale securities. During Year 2, these securities were sold at a loss equal to the unrealized loss previously recognized. The reclassification adjustment should include which of the following? A. The unrealized loss should be credited to the investment account. B. The unrealized loss should be credited to the other comprehensive income account. C. The unrealized loss should be debited to the other comprehensive income account. D. The unrealized loss should be credited to beginning retained earnings.

B. The unrealized loss should be credited to the other comprehensive income account. credit to OCI remove unrealize loss The related unrealized holding loss (losses are usually debits) on an available-for-sale security is a debit balance (lowering) in other comprehensive income (equity account, normal credit balance). When the asset is sold, the related accounts are removed, including this unrealized holding loss in other comprehensive income. A credit to other comprehensive income will remove it. The corresponding debit in the entry is to a realized loss.

What is the purpose of SFAC 4 as stated in that concepts statement? A. To provide the methods for preparing financial statements for nonbusiness entities B. To provide a basis for establishing detailed accounting and reporting standards for nonbusiness entities C. To provide detailed accounting and reporting standards for nonbusiness entities D. All of the answer choices are correct.

B. To provide a basis for establishing detailed accounting and reporting standards for nonbusiness entities SFAC 4, Objectives of Financial Reporting by Nonbusiness Organizations, represents the most recent expression of the overall purposes and related objectives of financial reporting by nonbusiness organizations. The purposes and related accounting and reporting objectives set forth in SFAC 4 are concepts—not standards—and are designed to provide a basis for establishing detailed accounting and reporting standards.

Which of the following conditions or events most likely would cause management to have substantial doubt about an entity's ability to continue as a going concern? A. Significant related party transactions are pervasive. B. Usual trade credit from suppliers is denied. C. Arrearage in preferred stock dividends is paid. D. Restrictions on the disposal of principal assets are present.

B. Usual trade credit from suppliers is denied. Management must evaluate whether there are conditions and events, considered in the aggregate, that raise substantial doubt about an entity's ability to continue as a going concern within one year after the date that the financial statements are issued. The denial of usual trade credit, in combination with other related business conditions, is not a good sign. The other items listed are not particularly indicative of going concern problems.

A company using the composite depreciation method for its fleet of trucks, cars, and campers retired one of its trucks and received cash from a salvage company. The net carrying amount of these composite asset accounts would be decreased by the: A. cash proceeds received and original cost of the truck. B. cash proceeds received. C. original cost of the truck less the cash proceeds. D. original cost of the truck.

B. cash proceeds received. When applying group or composite depreciation methods, when one sells an asset, the cost of the asset is removed, and the accumulated depreciation is assumed to be equal to the difference between cash received and cost. When the asset cost and this accumulated depreciation amount are both removed, the carrying amount of the asset accounts is decreased by the cash proceeds exactly.

A balance arising from the translation or remeasurement of a subsidiary's foreign currency financial statements is reported in the consolidated income statement when the subsidiary's functional currency is: A. neither the foreign currency nor the U.S. dollar. B. the U.S. dollar. C. the foreign currency. D. both the foreign currency and the U.S. dollar.

B. the U.S. dollar. The objective of translation or remeasurement is to report the subsidiary's income statement results in the U.S. parent's currency—which is the U.S. dollar.

On June 1 of the current year, a company entered into a real estate lease agreement for a new building. The lease is an operating lease and is fully executed on that day. According to the terms of the lease, payments of $28,900 per month are scheduled to begin on October 1 of the current year and to continue each month thereafter for 56 months. The lease term spans five years. The company has a calendar year-end. What amount is the company's lease expense for the current calendar year? A. $86,700 B. $161,838 C. $188,813 D. $202,300

C. $188,813 The inception of a lease is the date of the lease agreement. Rental expense should be as of that date. When the lease payments begin later than the inception date, the lease payments must be spread evenly over the longer period of time, which includes the months between the inception date and the beginning of the lease payments. $28,900 × 56 months = $1,618,400 $1,618,400 ÷ 60 months = $26,973.33 $26,973.33 × 7 months (June through December) = $188,813

A county's balances in the general fund included the following: Appropriations $435,000 Encumbrances 18,000 Expenditures 164,000 Vouchers payable 23,000 What is the remaining amount available for use by the county? A. $230,000 B. $248,000 C. $253,000 D. $271,000

C. $253,000 The appropriations included in the adopted budget of the general fund represent the maximum authorized for expenditure during the period. If $164,000 has already been expended and another $18,000 has been encumbered or committed, then only $253,000 remains available for spending. The vouchers payable represent past expenditures waiting only for cash payment.

On January 1, 2X01, Big Oil placed in service an offshore oil platform that it constructed. Big Oil is legally required to dismantle and remove the platform at the end of its 10-year estimated life. Using expected present value techniques, Big Oil recorded an estimated asset retirement obligation (ARO) of $100,000 on January 1, 2X01. The ARO measurements on January 1, 2X01, are as follows: Expected cash flow before inflation: $190,000 Expected cash flow adjusted for inflation and market risk: $220,000 Present value using credit-adjusted risk-free rate: $100,000 Assuming that the ARO is settled on December 31, 2X10, for $170,000, what is the gain or loss on the settlement? A. $70,000 loss B. $20,000 gain C. $50,000 gain D. No gain or loss

C. $50,000 gain The gain or loss on the settlement of the ARO liability is the difference between the ARO liability on settlement date of $220,000 and the actual settlement cost of $170,000. FASB ASC 410-20-40-2 requires the initial liability of $100,000 to be increased to the expected cash flow adjusted for market risk and inflation. Since the expected cash payment for the ARO liability was $220,000, a gain on settlement of $50,000 results.

At its date of incorporation, Glean, Inc., issued 100,000 shares of its $10 par common stock at $11 per share. During the current year, Glean acquired 30,000 shares of its common stock at a price of $16 per share and accounted for them by the cost method. Subsequently, these shares were reissued at a price of $12 per share. There have been no other issuances or acquisitions of its own common stock. What effect does the reissuance of the stock have on the following accounts? A. A decrease in both retained earnings and additional paid-in capital B. No effect on retained earnings and a decrease in additional paid-in capital C. A decrease in retained earnings and no effect on additional paid-in capital D. No effect on retained earnings or additional paid-in capital

C. A decrease in retained earnings and no effect on additional paid-in capital Using the cost method of accounting, additional paid in capital from treasury stock is credited when treasury stock is reissued at excess. The sale of the treasury stock resulted in a $120,000 loss $12 selling price - $16 cost to purchase = $4 30,000 shares x $4 = $120,000 The journal entry for reissuance of the treasury stock is as follows: Cash ($12 x 30,000) $360,000 Retained Earnings $120,000 Treasury Stock ($16 x 30,000) $480,000 There was only one treasury stock reissuance so there is no additional paid in capital transaction to be made. Therefore, under the cost method Glean would record the loss (decrease or debit) to retained earnings and there would be no effect on additional paid-in capital.

Which of the following statements is correct concerning financial reporting for a governmental entity? A. A combined statement of cash flows should be presented for proprietary and fiduciary funds. B. No statement of cash flows is required. C. A statement of cash flows should be presented for proprietary funds only. D. Either the direct or indirect method of presenting cash flows from operating activities may be used.

C. A statement of cash flows should be presented for proprietary funds only (enterprise or internal service) Governments should present a statement of cash flows for proprietary funds using the direct method of presenting cash flows from operating activities.

Falltown provides combining financial statements in its comprehensive financial report, detailing fiduciary funds aggregated in the basic financial statements. The combining financial statements for agency funds would include: A. a combining statement of fiduciary net position—agency funds. B. a combining statement of changes in fiduciary net position—agency funds. C. a combining statement of fiduciary net position—fiduciary funds. D. a combining statement of changes in fiduciary net position—fiduciary funds.

C. a combining statement of fiduciary net position—fiduciary funds. Fiduciary fund information should cover all fiduciary funds of a government with a separate column for each fiduciary fund type: pension trust funds, investment trust funds, private-purpose trust funds, and agency funds. A combining statement including all fiduciary funds would not be used. Further, the agency fund does not report a statement of changes in fiduciary net position. It consists only of assets and liabilities that are reported on the financial statement date in the statement of fiduciary net position.

Grid Corp. acquired some of its own common shares at a price greater than both their par value and original issue price but less than their book value. Grid uses the cost method of accounting for treasury stock. What is the impact of this acquisition on total stockholders' equity and the book value per common share? A. Increase in both total stockholders' equity and book value per share B. Increase in total stockholders' equity and decrease in book value per share C. Decrease in total stockholders' equity and increase in book value per share D. Decrease in both total stockholders' equity and book value per share

C. Decrease in total stockholders' equity and increase in book value per share T/S increase asset BV increase b/c divide by way less total equity but less shares = large num/small denom The entry to record the treasury stock is: Dr. Cr. Treasury stock XXX Cash XXX Treasury stock is a deduction from total stockholders' equity; therefore total stockholders' equity decreases (becomes asset). Since the price paid for the reacquired shares is less than their book value, the book value per share of the remaining outstanding shares must increase. Before the reacquisition of the shares, the book value per share is the same for all shares (i.e., those that will be reacquired and those that will not be). The reduction in total book value of the entity is equal to the total cost of the shares reacquired, which is less than the total book value of those shares. Thus, the percentage reduction in total book value was less than the percentage reduction in the number of outstanding shares. Therefore, the book value per share of the remaining shares must increase. For example, assume that an entity has total stockholders' equity of $1,000,000 and 10,000 shares outstanding. The book value is $1,000,000 ÷ 10,000 shares = $100. If 1,000 shares are reacquired at $60 per share (an amount less than the $100 book value per share), the entry would be: Dr. Cr. Treasury stock (1,000 x $60) 60,000 Cash 60,000 After the reacquisition of the shares, total stockholders' equity is $1,000,000 - $60,000, or $940,000. The book value per share of the remaining outstanding shares is $940,000 ÷ 9,000 outstanding shares, or $104.44. Therefore, the reacquisition of outstanding shares at a price less than their book value per share causes the book value per share of the remaining outstanding shares to increase.

When a loan receivable is impaired but foreclosure is not probable, which of the following may the creditor use to measure the impairment? The loan's observable market price The fair value of the collateral if the loan is collateral dependent A. I only B. II only C. Either I or II D. Neither I nor II

C. Either I or II Under the appropriate circumstances, both a market price of a debt or a realizable value of collateral available could be used to measure the impairment of a receivable. There are three ways to measure the present fair value of an impaired loan and they are listed in FASB ASC 310-10-35-22: 1. The present value of the expected future cash flows from the loan discounted at the loan's original effective rate 2. The amount the loan could be sold for 3. The net realizable value of the available loan collateral The difference between the loan's carrying value and its fair value is the loan impairment.

During 20X1, Sloan, Inc., began a project to construct new corporate headquarters. Sloan purchased land with an existing building for $750,000. The land was valued at $700,000 and the building at $50,000. Sloan planned to demolish the building and construct a new office building on the site. What is the appropriate accounting treatment for interest of $147,000 on construction financing incurred after completion of construction? A. Classify as land and do not depreciate B. Classify as building and depreciate C. Expense

C. Expense FASB ASC 835-20-25-5 provides that: The capitalization period shall end when the asset is substantially complete and ready for its intended use. Therefore, the $147,000 in construction financing incurred after completion of construction should be expensed.

Restorations of carrying value for long-lived assets are permitted if an asset's fair value increases subsequent (after) to recording an impairment loss for which of the following? A. Both held for use and held for disposal B. Held for use C. Held for disposal D. Neither held for use nor held for disposal

C. Held for disposal A long-lived asset classified as held for sale (disposal) must be measured at the lower of its carrying amount or fair value less cost to sell. A loss should be recognized for any initial or subsequent write-down to fair value less cost to sell. A gain should be recognized for any subsequent increase in fair value less cost to sell, but not in excess of the cumulative loss previously recognized for a write-down to fair value less cost to sell.

During Year 3, Gilman Co. purchased 5,000 shares of the 500,000 outstanding shares of Meteor Corp.'s common stock for $35,000. During Year 3, Gilman received $1,800 of dividends from its investment in Meteor's stock. The fair value of Gilman's investment on December 31, Year 3, is $32,000. Gilman has elected the fair value option for this investment. What amount of income or loss that is attributable to the Meteor stock investment should be reflected in Gilman's earnings for Year 3? A. Income of $4,800 B. Income of $1,800 C. Loss of $1,200 D. Loss of $3,000

C. Loss of $1,200 There is both a gain and a loss that must be recognized: Dividend $ 1,800 Decline in value ($35,000 - $32,000) (3,000) -------- Net loss $(1,200)

Which of the following transactions is an expenditure of a governmental unit's general fund? A. Contribution of enterprise fund capital by the general fund B. Operating subsidy transfer from the general fund to an enterprise fund C. Routine employer contributions from the general fund to a pension trust fund D. Transfer from the general fund to a capital projects fund

C. Routine employer contributions from the general fund to a pension trust fund GASB P20.113 requires employer governments to report employer pension contributions as an expenditure from a governmental fund such as the general fund. The other answer choices are examples of nonreciprocal interfund activity and are transfers that should be reported as other financing uses rather than as expenditures of the general fund. This is true regardless of whether the payee is a retirement system independent of the employer government or a pension trust fund that is part of the same government. In the latter case, the general fund contribution would be viewed as a reciprocal interfund activity, similar to an exchange transaction with an external entity. The other items are transfers from the general fund reported as "other financing uses."

The SEC's rulemaking procedures identified on their website include which of the following steps? A. Issue identification B. Commissions deliberation C. Rule adoption D. None of the answer choices are correct.

C. Rule adoption Concept Release: The rulemaking process usually begins with a rule proposal, but sometimes an issue is so unique and/or complicated that the SEC seeks out public input on which, if any, regulatory approach is appropriate. A concept release is issued describing the area of interest and the SEC's concerns, usually identifying different approaches to addressing the problem, followed by a series of questions that seek the views of the public on the issue. The public's feedback is taken into consideration as the SEC decides which approach, if any, is appropriate. Rule Proposal: The SEC publishes a detailed formal rule proposal for public comment. Unlike a concept release, a rule proposal advances specific objectives and methods for achieving them. Typically, the SEC provides between 30 and 60 days for review and comment. Just as with a concept release, the public comment is considered vital to the formulation of a final rule. Rule Adoption: Finally, the SEC Commissioners consider what they have learned from the public exposure of the proposed rule, and seek to agree on the specifics of a final rule. If a final measure is then adopted by vote of the full Commission, it becomes part of the official rules that govern the securities industry.

The letter of transmittal and the statistical section are classified as: A. basic financial statement. B. required supplementary schedule. C. other. D. All of the answer choices are correct.

C. other. The letter of transmittal and the statistical section required for a CAFR and GFOA's certificate are neither basic statements nor required supplemental information. Comprehensive Annual Financial Report (CAFR) is a set of U.S. government financial statements comprising the financial report of a state, municipal or other governmental entity that complies with the accounting requirements promulgated by the Governmental Accounting Standards Board (GASB).

A company that wishes to disclose information about the effect of changing prices should report this information in: A. the body of the financial statements. B. the notes to the financial statements. C. supplementary information to the financial statements. D. management's report to shareholders.

C. supplementary information to the financial statements. A business entity that prepares its financial statements in U.S. dollars and in accordance with U.S. generally accepted accounting principles is encouraged, but not required, to disclose supplementary information on the effects of changing prices.

The estimated revenues control account of a governmental unit is debited when: A. actual revenues are recorded. B. actual revenues are collected. C. the budget is recorded. D. the budget is closed at the end of the year.

C. the budget is recorded. Adoption of the operating budget is recorded in the general ledger of a governmental fund with a budgetary entry that debits estimated revenues and credits appropriations, with the budgetary fund balance account being debited or credited for the difference. The debit balance in estimated revenues is compared periodically with the credit balance in revenues to determine whether actual revenues are behind or ahead of the budgeted (estimated) amount.

In preparing its cash flow statement for the year ending December 31, 20X1, Reve Co. collected the following data: Gain on sale of equipment $ (6,000) Proceeds from sale of equipment 10,000 Purchase of A.S., Inc., bonds (par value $200,000) (180,000) Amortization of bond discount 2,000 Dividends declared (45,000) Dividends paid (38,000) Proceeds from sale of Treasury stock (carrying amount $65,000) 75,000 In its December 31, 20X1, statement of cash flows, what amount should Reve report as net cash provided by financing activities? A. $20,000 B. $27,000 C. $30,000 D. $37,000

Cash inflows from financing activities: Proceeds from sale of treasury stock $75,000 Cash outflows from financing activities: Dividends paid (38,000) -------- Net cash provided by financing activities $37,000 Dividends declared created a liability, but until they are paid, no cash flows out of the corporation.

Larkin Co. reported a taxable loss of $10,000 in 20X1, its first year of operations, and taxable income of $0 in 20X2. Larkin had no temporary or permanent differences in either 20X1 or 20X2. At the end of 20X1 Larkin believed that 30% of the operating loss carryforward would not be realized; therefore, a valuation allowance of $1,200 (30% of $10,000 NOL × 40% tax rate) was necessary. At the end of 20X2, Larkin believes that the valuation allowance is no longer necessary. Assuming a tax rate of 40%, Larkin should report total income tax expense (benefit) in 20X1 and 20X2 of: A. $0 in 20X1 and $0 in 20X2. B. $(4,000) in 20X1 and $0 in 20X2. C. $(2,800) in 20X1 and $0 in 20X2. D. $(2,800) in 20X1 and $(1,200) in 20X2.

D. $(2,800) in 20X1 and $(1,200) in 20X2. In 20X1, Larkin should recognize a deferred tax asset and the related deferred tax benefit of $4,000 ($10,000 NOL × 40% tax rate). However, Larkin also must recognize a $1,200 valuation allowance in 20X1. Thus, in 20X1 Larkin should recognize a net tax expense (benefit) of $(4,000) + $1,200 = $(2,800). Note that the recognition of the valuation allowance reduces the net tax benefit recognized in 20X1. The decision in 20X2 that the valuation allowance is no longer necessary means that the valuation allowance should be eliminated, as shown in the following entry in 20X2: Valuation allowance $1,200 Tax expense/benefit - deferred $1,200 Therefore, tax expense (benefit) in 20X2 has a credit balance of $(1,200), indicating a deferred tax benefit. This $(1,200) tax benefit recognized in 20X2 is the change in deferred tax expense/benefit arising from changed circumstances causing a change in judgment as to the amount of valuation.

RST Charities received equities securities valued at $100,000 as an unrestricted gift. During the year, RST received $5,000 in dividends from these securities; at year-end, the securities had a fair market value of $110,000. By what amount did these transactions increase RST's net assets? A. $100,000 B. $105,000 C. $110,000 D. $115,000

D. $115,000 Investments are initially recorded at fair value if received as a contribution or gift. Unrealized gains on investments carried at fair value also increase net assets. As the investments themselves were an unrestricted gift, the unrealized gain would increase unrestricted net assets. Investment income includes dividends that increase unrestricted net assets unless there are donor stipulations.

Ball Corp. had the following foreign currency transactions during the current year: Goods purchased from a foreign supplier on January 20 for the U.S. dollar equivalent of $90,000. The invoice was paid on March 20, at the U.S. dollar equivalent of $96,000. On July 1, Ball borrowed the U.S. dollar equivalent of $500,000 evidenced by a note that was payable in the lender's local currency on July 1, in two years. On December 31, the U.S. dollar equivalents of the principal amount and accrued interest were $520,000 and $26,000, respectively. Interest on the note is 10% per annum. In Ball's year-end income statement, what amount should be included as foreign exchange loss? A. $0 B. $6,000 C. $21,000 D. $27,000

D. $27,000 Ball's year-end income statement should include $27,000 as foreign exchange loss, calculated as follows: Foreign curr loss goods purchased ($90k- $96k) $ 6,000 Loan principal foreign curry loss($500k - $520k) 20,000 Loan interest foreign curr loss ($25k* - $26k) 1,000 Total loss $27,000 * Interest in U.S. dollars: $500,000 × 0.10 × 1/2 year = $25,000

On January 1, Feld traded a delivery truck and paid $10,000 cash for a tow truck owned by Baker. The delivery truck had an original cost of $140,000, accumulated depreciation of $80,000, and an estimated fair value of $90,000. Feld estimated the fair value of Baker's tow truck to be $100,000. The transaction had commercial substance. What amount of gain should be recognized by Feld? A. $0 B. $3,000 C. $10,000 D. $30,000

D. $30,000 The gain is always FV of old asset - CV of old asset. Feld should recognize $30,000 gain: Value received ($100,000 value of tow truck less $10,000) $90,000 Book value of delivery truck: Cost $140,000 Accumulated depreciation 80,000 60,000 ------- Gain $30,000 Exchanges that have commercial substance (future cash flows are expected to change) should be accounted for at fair value

Young Co. issues $800,000 of 10% bonds dated January 1, Year 1. Interest is payable semiannually on June 30 and December 31. The bonds mature in five years. The current market for similar bonds is 8%. The entire issue is sold on the date of issue. The following values are given: Present Value of Ordinary Annuity Present Value of $1 ---------------- ------------------- N=10; i=0.04 8.11090 0.67556 N=10; i=0.05 7.72173 0.61391 What amount of proceeds on the sale of bonds should Young report? A. $799,997 B. $815,564 C. $849,317 D. $864,884

D. $864,884 This question is about the computation of the issue price for a bond. The bonds will pay semiannually, and thus will pay $40,000 twice each year, computed as follows: Face amount of $800,000 × 10% coupon × 6/12 (half-year) = $40,000 The yield of the bonds is 8% annually, but in half-year periods it is 4% a half-year. The present value of the bonds is thus the $40,000 multiplied by the present value of the ordinary annuity for 10 periods and 4%, plus the $800,000 par value of the bonds multiplied by the present value of $1 at 10 periods, 4%: Issue price = ($40,000 × 8.11090) + ($800,000 × 0.67556) = $324,436 + $540,448 = $864,884

Band Co. uses the equity method to account for its investment in Guard, Inc., common stock. How should Band record a 2% stock dividend received from Guard? A. As a dividend revenue at Guard's carrying value of the stock B. As dividend revenue at the market value of the stock C. As a reduction in the total cost of Guard stock owned D. As a memorandum entry reducing the unit cost of all Guard stock owned

D. As a memorandum entry reducing the unit cost of all Guard stock owned A company using the equity method to account for an investment does not recognize dividends received as revenue. When a cash dividend is received, the receipt of cash is treated as a liquidation of the investment and the carrying amount of the investment is reduced by the amount of the dividend. However, when additional stock shares are received in lieu of cash, no liquidation of the investment has occurred. Instead, the investment carrying value now applies to a larger number of shares held by the investor. Therefore, the investor needs only to note that the value per share of its investment has decreased and the number of shares has increased.

Grove Township issued $50,000 of bond anticipation notes at face amount in the current year and placed the proceeds into its capital projects fund. All legal steps were taken to refinance the notes, but Grove was unable to consummate refinancing. In the capital projects fund, what account should be credited to record the $50,000 proceeds? A. Other Financing Sources Control B. Revenues Control C. Deferred Revenues D. Bond Anticipation Notes Payable

D. Bond Anticipation Notes Payable Bond anticipation notes are included as general long-term liabilities. The amount would never be revenue or a deferred revenue as the amount is due to be repaid at some point.

Government C sponsors a public entity risk pool in which Government C is also a participant. However, Government C is not the predominant participant in the pool. For the situation described, indicate the fund(s) Government C must use to account for the risk pool. A. Agency B. Special revenue C. Internal service D. Enterprise

D. Enterprise The proper accounting and financial reporting for this risk pool depend on whether the sponsoring government is also the predominant participant. If Government C is not the predominant participant, the pool would be treated as a stand-alone pool in substance and thus be accounted for in an enterprise fund and Government C's participation in the pool would be considered incidental

Which of the following statements meet the measurement and recognition criteria for landfill closure and post-closure costs? A. Landfills should only be accounted for in the general fund. B. Total landfill liabilities should be recognized in the general long-term debt account group. C. Expense recognition should begin when waste is accepted and should continue through the post-closure period. D. Equipment and facilities included in estimated total current cost of closure and post-closure care should not be reported as capital assets.

D. Equipment and facilities included in estimated total current cost of closure and post-closure care should not be reported as capital assets. Equipment and facilities included in the estimated total current cost of closure and post-closure care should not be reported as capital assets. Equipment, facilities, services, and final cover, included in the estimated total current cost, should be reported as a reduction of the accrued liability for landfill closure and post-closure care, when they are acquired. GASB L10.107

Under IFRS, which of the following measurements is allowed to estimate and report the liability for the cost of settling a lawsuit? A. Estimate only the smallest item in the estimated range of losses B. Estimate only the best estimate to settle C. Discount amounts of estimated loss to present value D. Estimate only the best estimate to settle and discount amounts of estimated loss to present value

D. Estimate only the best estimate to settle and discount amounts of estimated loss to present value When a range of amounts that may be lost in a lawsuit are established, the best number in the range must be chosen to accrue. The chosen amount must always be discounted to present value.

An entity purchased new machinery from a supplier before the entity's year-end. The entity paid freight charges for the purchased machinery. The entity took out a loan from a bank to finance the purchase. Under IFRS, what is the proper accounting treatment for the freight and interest costs related to the machinery purchase? A. The freight and interest costs should be immediately expensed. B. The freight and interest costs should be capitalized as part of property, plant, and equipment. C. The interest cost should be capitalized as part of property, plant, and equipment, and the freight cost should be immediately expensed. D. The freight cost should be capitalized as part of property, plant, and equipment, and the interest cost should be immediately expensed.

D. The freight cost should be capitalized as part of property, plant, and equipment, and the interest cost should be immediately expensed. The costs to buy equipment, along with the costs to bring it to its location for use and make it ready for use, are capitalized into the cost of the equipment. Any interest costs in financing the purchase of equipment (which is otherwise ready to use) are finance (interest) costs and are expensed.

GASB 1600.103 requires governmental entities to issue which two sets of financial statements? A. The balance sheet and the statement of owner's equity B. The income statement and the statement of cash flows C. The statement of net position and the statement of cash flows D. The statement of net position and the statement of activities

D. The statement of net position and the statement of activities B/S and I/S Since budgetary compliance requires governmental entities to prepare cash-basis budgets, GASB 1600.103 requires the issuance of two financial statements: The balance sheet or statement of net position The statement of revenue and expenses or statement of activities

Which of the following phrases best describes a Level 1 input for measuring the fair value of an asset or liability? A. Inputs for the asset or liability based on the reporting entity's internal data B. Quoted prices for similar assets or liabilities in active markets C. Inputs that are principally derived from or corroborated by observable market data D. Unadjusted quoted prices for identical assets or liabilities in active markets

D. Unadjusted quoted prices for identical assets or liabilities in active markets level 2: inputs other than quoted prices included within Level 1 that are observable for similar assets or liabilities, either directly or indirectly level 3: unobservable inputs for the asset or liability

Louisiana Designer Yarn, Inc., applies IFRS and does substantial research and development work in designing new processes to produce its products. One yarn-producing machine design, which is in an advanced stage of development, and which the company thinks its present prototype model should be both technologically feasible and affordable to produce, is still going to be developed, internally, for 18 months prior to being finished. Can the corporation recognize and capitalize any of the costs of developing the new machine design? A. No, because all research and development costs are expensed as incurred B. No, because the machine design is an internally developed intangible asset, with no purchase transaction C. Yes, but only if the machine is to be used by the corporation itself for internal operations and production D. Yes, as long as the design is likely to be feasible and marketable or profitable to use internally for future production

D. Yes, as long as the design is likely to be feasible and marketable or profitable to use internally for future production Once a development project reaches the stage of a working model or prototype, and is found to be technologically feasible and financially affordable to complete, then it can be capitalized, and additional development costs added to its cost on the company books. The asset can be intended for sale or internal use, so long as it is expected to be valuable for that purpose.

A business interest that constitutes a large part of an individual's total assets should be presented in a personal statement of financial condition as: A. a separate listing of the individual assets and liabilities at cost. B. separate line items of both total assets and total liabilities at cost. C. a single amount equal to the proprietorship equity. D. a single amount equal to the estimated current value of the business interest.

D. a single amount equal to the estimated current value of the business interest. FASB ASC 274-10-45-9, Accounting and Financial Reporting for Personal Financial Statements, contains guidelines for preparation of personal statements of financial condition. For business interests that constitute a large part of a person's total assets: The estimated current value of an investment...should be shown in one amount as an investment....

For a capital lease, the amount recorded initially by the lessee as a liability should normally: A. exceed the total of the minimum lease payments. B. exceed the present value of the minimum lease payments at the beginning of the lease. C. equal the total of the minimum lease payments. D. equal the present value of the minimum lease payments at the beginning of the lease.

D. equal the present value of the minimum lease payments at the beginning of the lease. The lessee shall measure a capital lease asset and capital lease obligation initially at an amount equal to the present value at the beginning of the lease term of minimum lease payments during the lease term.

Each of the following events is required to be reported to the United States Securities and Exchange Commission on Form 8-K, except: A. the creation of an obligation under an off-balance sheet arrangement of a registrant. B. the unregistered sale of equity securities. C. a change in a registrant's certifying accountant. D. the quarterly results of operations and financial condition of a registrant.

D. the quarterly results of operations and financial condition of a registrant. Public companies must report certain material corporate events on a more current basis than quarterly (on Form 10-Q) or annually (on Form 10-K). Form 8-K reports current events, allowing investors to obtain information in a timely manner about events material to a company's performance. Some of the most common events reported on Form 8-K include: 1. a change in a registrant's certifying accountant 2. notification about entering into material agreements (including merger agreements and sales of securities) 3. entering into debt or other direct financial obligations. Companies have four business days from the event date to file a Form 8-K.

KLU Broadcast Co. entered into an agreement to exchange unsold advertising time for travel and lodging services with Hotel Co. As of June 30, travel and lodging services of $10,000 were used by KLU. However, the advertising service had not been provided. How should KLU account for travel and lodging in its June 30 financial statements? A. Revenue and expense is recognized when the agreement is complete. B. An asset and revenue for $10,000 is recognized. C. An expense and liability of $10,000 is recognized. D. Not reported

KLU has incurred expenses for travel and lodging and has a corresponding liability for unearned revenue. The revenue from providing advertising time is not earned and cannot be recognized as revenue until the advertising time actually has been provided for Hotel Co.

Which of the following financial instruments must be presented between the liabilities section and the equity section? A. Mandatorily redeemable financial instruments B. Obligations to repurchase the issuer's equity shares by transferring assets C. Certain obligations to issue a variable number of shares D. None of the answer choices are correct; all of these financial instruments must be presented as liabilities.

FASB ASC 480-10-10 addresses the classification of the following three classes of freestanding financial instruments: Mandatorily redeemable financial instruments Obligations to repurchase the issuer's equity shares by transferring assets Certain obligations to issue a variable number of shares These three classes of financial instruments must be presented in the balance sheet as liabilities. They may not be presented between the liabilities section and the equity section.

The presentation of financial statements must be applied within an identifiable framework (AU-C 800.A15-.A18). Normally, the framework is provided by generally accepted accounting principles (GAAP). However, in some circumstances, a different framework may be used. Which of the following would not be indicative of an acceptable framework for the presentation of financial statements? A. Reporting on cash basis but capitalizing fixed assets and recording depreciation B. Ignoring accrued income and expenses C. Measuring inflation and reporting it on the financial statements D. Modifying items on the cash flow statement based on definite criteria

Modifying items on the cash flow statement based on definite criteria Financial statements may be prepared using a comprehensive basis of accounting other than generally accepted accounting principles (often referred to as a special purpose framework). Examples given by AU-C 800.07 are as follows: Cash basis. A basis of accounting that the entity uses to record cash receipts and disbursements and modifications of the cash basis having substantial support (for example, recording depreciation on fixed assets). Tax basis. A basis of accounting that the entity uses to file its income tax return for the period covered by the financial statements. Regulatory basis. A basis of accounting that the entity uses to comply with the requirements or financial reporting provisions of a regulatory agency to whose jurisdiction the entity is subject (for example, a basis of accounting that insurance companies use pursuant to the accounting practices prescribed or permitted by a state insurance commission). Contractual basis. A basis of accounting that the entity uses to comply with an agreement between the entity and one or more third parties other than the auditor.

On January 30, 20X2, Edge issued $10,000,000 bonds at a premium of $500,000. Adjustment of A. 500k B. 100k C. 0

No adjustment The bonds were issued after the balance sheet date and would therefore not be reported on the balance sheet; however, the matter should be disclosed in the notes to the financial statements to prevent them from being misleading (FASB ASC 450-20-25-6).

On January 5, 20X2, a warehouse containing a substantial portion of Edge's inventory was destroyed by fire. Edge expects to recover the entire loss, except for a $250,000 deductible, from insurance. Adjustment of A. 250k B. 0

No adjustment The loss did not occur until after the balance sheet date, therefore the financial statements would not be adjusted. The notes to the financial statements should disclose the $250,000 loss arising from this subsequent event (FASB ASC 450-20-25-6).

The following information pertains to Spee Co.'s 20X1 sales: Cash Sales Gross $40,000 Returns and allowances 2,000 Credit Sales Gross 60,000 Discounts 3,000 On January 1, 20X1, customers owed Spee $20,000. On December 31, 20X1, customers owed Spee $15,000. Spee uses the direct write-off method for bad debts. No bad debts were recorded in 20X1. Under the cash basis of accounting, what amount of revenue should Spee report for 20X1? A. $100,000 B. $95,000 C. $85,000 D. $38,000

One needs to convert from accrual to cash method income (revenue when collected in cash). Cash collect cash sale($40k- 2k) =$ 38,000 Cash collected from credit sales: Net cred sales forX1 ($60k - 3k) $57,000 January 1, 20X1, A/R 20,000 Less Dec 31, 20X1, A/R (15,000) ------- -------- Cash basis revenue for 20X1 $100,000

Topic 275 of the FASB's Accounting Standards Codification is entitled "Risks and Uncertainties." The primary subject discussed in this topic is: A. bankruptcy. B. going concern. C. disclosure. D. All of the answer choices are discussed.

One of the purposes of financial statements is to provide information to help users predict the reporting entity's future cash flows and results of operations. This assessment depends, to some degree, on the users' knowledge and assessment of the risks and uncertainties involving the entity's operations. Disclosure of these risks and uncertainties is a critical component of the user's process of evaluating these variables. FASB ASC 275-10 addresses the disclosures required to facilitate a user's evaluation of an entity's risks and uncertainties.

The following condensed balance sheet is presented for the partnership of Alfa and Beda, who share profits and losses in the ratio of 60:40, respectively: Cash $ 45,000 Other assets 625,000 Beda (loan) 30,000 -------- $700,000 ======== Accounts payable $120,000 Alfa (capital) 348,000 Beda (capital) 232,000 -------- $700,000 ======== Instead of admitting a new partner, Alfa and Beda decide to liquidate the partnership. If the other assets are sold for $500,000, what amount of the available cash should be distributed to Alfa? A. $255,000 B. $273,000 C. $327,000 D. $348,000

Prior to any liquidation computations, combine the partner's loans to the partnership with their capital account balances. Then pay the debts, sell the assets (and account for losses on sales), divide up the losses, and then distribute the remaining cash based on the remaining partner capital account balances. Loss from sale of other assets = $625,000 - $500,000 = $125,000 Allocation of loss to Alfa = .60 x $125,000 = $ 75,000 Alfa's new capital balance = $348,000 - $75,000 = $273,000 Alfa should receive $273,000 of cash.

In its December 31 balance sheet, Butler Co. reported trade accounts receivable of $250,000 and related allowance for uncollectible accounts of $20,000. What is the total amount of risk of accounting loss related to Butler's trade accounts receivable, and what amount of that risk is off-balance sheet risk? A. Risk of accounting loss: $0; Off-balance sheet risk: $0 B. Risk of accounting loss: $230,000; Off-balance sheet risk: $0 C. Risk of accounting loss: $230,000; Off-balance sheet risk: $20,000 D. Risk of accounting loss: $250,000; Off-balance sheet risk: $20,000

Risk of accounting loss: $230,000; Off-balance sheet risk: $0 FASB ASC 825-10-50-20 defines risk of accounting loss as the amount of write-off that a company would record if any party to an agreement failed to fully perform in accordance with the terms of the contract. Off-balance sheet risk occurs when the amount of an accounting loss exceeds the amount of the associated asset or liability recorded on the balance sheet. The maximum possible accounting loss associated with trade accounts receivable occurs if no amount of the current asset is collected. In this case, Butler's trade accounts receivable has a net book value of $230,000, which represents the maximum amount of potential write-off associated with trade accounts receivable. Butler would not be required to pay an amount in addition to the net book value of this asset, so there is no off-balance sheet risk.

Webb Co. has outstanding a 7%, 10-year $100,000 face-value bond. The bond was originally sold to yield 6% annual interest. Webb uses the effective interest rate method to amortize bond premium. On June 30, Year 2, the carrying amount of the outstanding bond was $105,000. What amount of unamortized premium on bond should Webb report in its June 30, Year 3, balance sheet? A. $1,050 B. $3,950 C. $4,300 D. $4,500

The interest paid for the year from June 30, Year 2. to June 30, Year 3. is based on the face amount ($100,000) multiplied by the stated 7% payment rate: $100,000 × 0.07 = $7,000 The interest expense using the interest method is based on the carrying amount of the debt multiplied by the yield of the debt: $105,000 × 0.06 = $6,300 The premium amortized from June 30, Year 2, to June 30, Year 3, is the difference of these two amounts: $7,000 - $6,300 = $700 Thus, the premium of $5,000 on June 30, Year 2, ($105,000 - $100,000, carrying amount less face amount) is lowered by the $700 premium amortization down to $4,300: $5,000 - $700 = $4,300

Karr, Inc., reported net income of $300,000 for 20X1. Changes occurred in several balance sheet accounts as follows: Equipment $25,000 increase Accumulated depreciation 40,000 increase Note payable 30,000 increase During 20X1, Karr sold equipment costing $25,000, with accumulated depreciation of $12,000, for a gain of $5,000. In December 20X1, Karr purchased equipment costing $50,000 with $20,000 cash and a 12% note payable of $30,000. Depreciation expense for the year was $52,000. In Karr's 20X1 statement of cash flows, net cash provided by operating activities should be: A. $340,000. B. $347,000. C. $352,000. D. $357,000.

Using the indirect method, Karr computes cash flow from operating activities as follows: Reported 20X1 net income $300,000 Add depreciation expense 52,000 Deduct gain on sale of equipment ( 5,000) --------- Net Cash flow from operating activities $347,000

On July 1, Year 1, Kay Corp. sold equipment to Mando Co. for $100,000. Kay accepted a 10% note receivable for the entire sales price. This note is payable in two equal installments of $50,000 plus accrued interest on December 31, Year 1 and Year 2. On July 1, Year 2, Kay discounted the note at a bank at an interest rate of 12%. Kay's proceeds from the discounted note were: A. $48,400. B. $49,350. C. $50,350. D. $51,700.

When accounting for a discounted note and computing the cash proceeds, one must first find the maturity value of the note, what will be received by the holder of the note when it comes due. By the time of the discounting, some of the principal has already been paid. Only the second installment, the final $50,000 principal plus interest, will be paid to the bank when due. At the end of December, Year 2, the $50,000 will be received by the bank along with 10% interest (since the principal will have been outstanding for a whole year). On December 31, Year 2, a total of $55,000 maturity value will be due: $50,000 + ($50,000 × 0.1) = $55,000 The discounted proceeds will be based on this amount, the discount rate (0.12), and the discounting period (from July to December of Year 2, 6 months). The discount amount is thus: $55,000 × 0.12 × 6/12 = $3,300 The cash proceeds are the maturity value less the discount: $55,000 - $3,300 = $51,700

A 15-year bond was issued in Year 1 at a discount. During Year 11, a 10-year bond was issued at face amount with the proceeds used to retire the 15-year bond at its face amount. The net effect of the Year 11 bond transactions was to increase long-term liabilities by the excess of the 10-year bond's face amount over the 15-year bond's: A. face amount. B. carrying amount. C. face amount less the deferred loss on bond retirement. D. carrying amount less the deferred loss on bond retirement.

carrying amount In order to solve this problem, give the 15-year bonds a face amount, say $100,000. If the bond was issued at a discount, then it was issued and was carried as a debt at a value below $100,000, say $90,000 (if the remaining unamortized discount was $10,000). If the 10-year bond was issued at its face amount, then it would be carried as a debt at its face amount. If the 10-year bond was issued during Year 11, then the 15-year bond was still a long-term debt, and the new 10-year bond would also be a long-term debt. If the 10-year bond was issued at face and the proceeds paid off the face amount of the 15-year bond, then the 10-year bond would have needed to be at least the face amount of the 15-year bond (they would have the same face amount). Thus, the new debt would be carried at $100,000, the old debt would be carried at below $100,000 (less the remaining unamortized discount), and the total long-term liabilities would be increased by the discount left, the amount the new 10-year debt carrying value is higher than the carrying value of the 15-year debt.


Ensembles d'études connexes

Fluid and Electrolyte SAC Mobility

View Set

Life Insurance Policy Provisions, Options, & Riders

View Set

Chapter 5 quiz Political science

View Set

ECON - Ch5.1- Chapter 5 - Banking - Section 1 - Financial Services & Institutions

View Set